Está en la página 1de 127

Applied Arithmetic

In any math problem on any aptitude test the question is actually covered in a
mask of situation. You interpret the it and formulized to an arithmetic expression.
Think; if you understand the situation and formulate the expression correctly but
make mistakes in calculating the value. You answer will be wrong. So the
arithmetic calculations are the most important to answer correctly.
In this module, each topic and method of solution is presented and explained
with emphasis on quick solutions.

Levels in Algebra Lessons

 Applied Arithmetic

 Algebra

 Applied Algebra

 Problem Solving Basic

 P.S. Intermediate

 Geometry

What is the Question?

Read the problem carefully. Make sure you answer the actual question. Many
time a student gives an answer in miles when the question asks for miles per
hour. Nobody sees your calculations but you. Only the correct answer counts. Be
careful.
Avoid Advanced Mathematics

Most often students having passed 12 grade with mathematics try to involve the
formulas they have learnt in college. For instance; to find the part of
circumference of a circle don't use S = rθ.

Lesson: Applied Arithmetic - 01

Applied Arithmetic Basic to Advance


Topics
[Page 1 of 36]

This module will review the following concepts and examine how they may be
tested on standardized exams:

 Percent Change
 Ratios
 Rates
 Averages
 Median, Mode, and Range
 Factorials
 Probability
 Combinations and Permutations

Video Lessons

Video lessons are available for each coming topic. Video discussion enhances
your understanding of the topic.
Applied Arithmetic problems tend to show up more frequently if you do well on
the quantitative section of a CAT. The good news about applied arithmetic
problems is that there are specific concepts that are always tested, and they are
tested in predictable ways. We'll show you what these concepts are, and how
they look on standardized exams. With a little practice you will be recognizing
and solving applied arithmetic problems with ease.
Lesson: Applied Arithmetic - 02

Percents
[Page 2 of 36]

Percent means out of 100. We can use the percent symbol (%) as a handy way
to write a fraction with a common denominator of 100. For example, instead of
saying "8 out of every 100 professional basketball players are female," we can
say "8% of professional basketball players are female." A percent can always be
written as a decimal, and a decimal can be written as a percent, by moving the
decimal point two places to the right.

Most "applied arithmetic" concepts are variations of fractions.


The key to doing well on applied arithmetic questions is knowing how to set up
the fractions.
Percents, as you probably know by now, are another way to express fractions.
"Percent" just means "over 100." So to convert a percent into a fraction, just drop
the % sign, place it over 100, and reduce the resulting fraction. To convert a
fraction into a percent, multiply the fraction by 100 and add the percent sign.
So far, no problem. But now let's check out a more challenging type of percent
question that you're sure to see on your exam.

Lesson: Applied Arithmetic - 03

Percent Change
[Page 3 of 36]

Percentage Change is all about comparing old to new values.

 Step 1: Calculate the change (subtract old value from the new value)
 Step 2: Divide that change by the old value (you will get a decimal number)
 Step 3: Convert that to a percentage (by multiplying by 100 and adding a
"%" sign)

Note: if the new value is greater then the old value, it is a percentage increase,
otherwise it is a decrease.
Video lesson on percent decrease

Where test-takers tend to get confused (as the test-maker well knows) is
on Percent Change problems. However, these problems can be easy points for
you, once you know how to set them up.

Percent Change = .
If you're ever confused about what the "original" amount is, note:

Percent increase (or greater) =

Percent decrease (or less) =

Video Lesson on Percent Increase

Lesson: Applied Arithmetic - 04

Apply The Percent Change Formula


[Page 4 of 36]

Video Lesson on Percentage Change Formula

Let's use the percent change formula to answer some questions.


Type your answer to each question into the Text Box, and
then click Continue.
1. If the student body at College X goes from 10,000 to 16,000, this
represents an increase of %.
1. If the student body at College X goes from 10,000 to 16,000, this
60
represents an increase of %.

Amount of increase = 6,000


Original (the smaller) amount = 10,000

If the population of College X then goes from 16,000 to 10,000, this


2.
represents a decrease of %.
If the population of College X then goes from 16,000 to 10,000, this
37.5
2. represents a decrease of %.

Amount of decrease = 6,000


Original (the larger) amount = 16,000

Lesson: Applied Arithmetic - 05

Percent Change Questions In-Format


Now try an in-format question. Click on the correct response and
then click Continue.
The retail price of a camera is discounted by the manufacturer by 20 percent.
The new price of the camera is then discounted by another 20 percent during a
store-wide sale. The original retail price of the camera was approximately what
percent greater than the final sale price?
36%
40%
56%
60%
64%

There are no actual values given in this percent problem, so we'll begin by
assuming that the original retail price of the camera was $100. So if that price is
discounted by 20 percent, the new price of the camera is $80. If that price is then
discounted by 20 percent, the final sale price of the camera is $80 - $16 = $64
(since 20% of 80 = 16).
Now read the question carefully: it asks what percent "greater" is the original
retail price than the final sale price, so set up the numbers carefully:

Percent greater =

Lesson: Applied Arithmetic - 06

Ratios
[Page 6 of 36]

We use ratios to make comparisons between two things. When we express ratios
in words, we use the word "to" -- we say "the ratio of something to something
else". Ratios can be written in several different ways.

Video Lesson - What is Ration

Ratios, like fractions, are another way of expressing Part/Wholerelationships,


although ratios often refer to Part/Part relationships as well. In either case, the
setup remains the same:

Classic Ratio Setup: What is the ratio


For example:
A class has 12 male students and 21 female students.

The ratio of male students to female students =

The ratio of male students to all the students in the class


Ratios can be expressed in several different ways:

 There are 4 male students for every 7 female students.


 The ratio of male students to female students is 4 : 7.

 The ratio of male students to female students is


Lesson: Applied Arithmetic - 07

Ratios, continued
[Page 7 of 36]

Video Lesson - What is Ratio

Use the classic ratio setup to answer the following questions. Type
each answer into the Text Box provided, and then click Continue.
At a Halloween party, there are 30 guests, 16 of whom are Draculas and 14 of
whom are Mummies.

1. What is the ratio of Draculas to Mummies at the party?

Answer: 8:7

At a Halloween party, there are 30 guests, 16 of whom are Draculas and 14 of


whom are Mummies.

1. What is the ratio of Draculas to Mummies at the party?

Answer: 8:7

2. What is the ratio of Mummies to the total number of party guests?


Answer: 7:15

Assuming no Mummies leave, how many Draculas need to arrive to make the
ratio of Draculas to guests 2:3?
3.
Answer: 12

Look to the number that doesn't change, and use that with your new ratio. The
number of Mummies remains the same at 14. You are told that you want

, so you want 2 Draculas for every 1 Mummy,

which means you want . Since , you want a total of 28


Draculas. There are presently 16 Draculas, so you want 12 more to arrive.

Lesson: Applied Arithmetic - 08

In-Format Ratio Problems


[Page 8 of 36]

Now try some in-format problems. Click on the correct


response and then click Continue.

Three lottery winners decide to split a cash prize in the ratio 1 : 2 : 3. If the cash
prize is $12,000, what is the greatest amount earned by one of the three
winners?
$1,000

$2,000
$3,000

$4,000

$6,000

The textbook approach is to come up with the ratio total by adding together

the ratio parts: 1 + 2 + 3 = 6, so the ratio for the biggest winner is the

entire prize of $12,000, so he or she would earn .


You could also try out the answer choices, starting with the number in the middle.
The greatest amount earned by one of the three winners is $3,000, and the ratio
of winnings is 1 : 2 : 3, so the money would be divided $1,000 : $2,000 : $3,000,
for a total prize of $6,000. This is half the actual amount of $12,000, so it looks
like $6,000 is the answer:
Let's prove it:
1 : 2 : 3 = $2,000 : $4,000 : $6,000, for a total of $12,000, which works.
$1,000
$2,000
$3,000
$4,000
$6,000

Lesson: Applied Arithmetic - 09

In-Format Ratio Problems, continued


[Page 9 of 36]

Let's try another. Click on the correct response and then


click Continue.

A cookie jar contains only two types of cookies—macaroons and ginger snaps.
The ratio of macaroons to ginger snaps is 2 to 5. Which of the following could be
the total number of cookies in the cookie jar?
42

45

48

52

57

If the ratio of the parts is 2 : 5, then the ratio total = 2 + 5 = 7. Thus the total
number of cookies must be a multiple of 7. The only answer choice that's a
multiple of 7 is Choice (1), 42 cookies.

Lesson: Applied Arithmetic - 10

Introducing Rates
[Page 10 of 36]

A rate is a ratio that compares two different kinds of numbers, such as miles per
hour or dollars per pound. A unit rate compares a quantity to its unit of measure.
A unit price is a rate comparing the price of an item to its unit of measure.
The rate "miles per hour" gives distance traveled per unit of time. Problems using
this type of rate can be solved using a proportion, or a formula.

Video Lesson - rates and Ratios

A rate is simply a ratio that compares distance over time, or amountover time,
or cost over units.

In other words: or or
The key to solving most Rate problems is to set them up as proportions, convert
units if necessary, and solve for the unknown value:
Solving Rates: The Textbook Approach
A waitress serves 3 diners every 5 minutes. At this rate, how many customers
will she serve in one hour?

Set up a proportion.
Convert units.

Cross-multiply and solve.

Practice Rate Problems


[Page 11 of 36]

Use the Textbook approach to answer the following


question. Type the answer into the field provided, and then
click Continue.

If gold braid costs 7 cents per inch, what is the maximum number of feet of gold
braid that could be purchased for $6.00 (assuming no tax)?
Answer: 7.14

So one could purchase 7 whole feet of gold braid.

In-Format Rate Problems


[Page 12 of 36]

Click on the correct response and then click Continue.


A copier can make 2.5 copies per second. At this rate, how many minutes would
it take to make 4,500 copies?
20

25

30

35

40

Set up a proportion.

Convert the units.

Cross-multiply and solve.

In-Format Rate Problems, continued


[Page 13 of 36]

Let's try another. Click on the correct response and then


click Continue.

Vaughn drove from his house to work at an average speed of 25 miles per hour
and then returned home along the same route at an average speed of 35 miles
per hour. If his total driving time for the trip was 2 hours, how many minutes did it
take Vaughn to drive from his house to work?
25

35

50

60
70

This is a tough rate problem, so rather than try to calculate the solution, you may
be better off trying to guesstimate the answer. For instance, it took Vaughn 2
hours to complete the round trip, and he drove moreslowly on the way to work
than on the return trip. Therefore, he must have spent more time going to work
than coming back. So the answer must be more than 60 minutes, and the only
possible answer choice that works is Choice (5), 70 minutes.
To calculate the answer mathematically, you could begin by calling the distance
either way d. Since Time = Distance/Rate, we can come up with the equation for
the time spent traveling and solve for d (keeping our units in hours for now):

. So . Thus , and

(We'll do what reducing we can and save the final number-


crunching for later). Now that we have the distance, we should divide that by the
rate (25 m.p.h.) to determine the time (in hours) it took Vaughn to drive to work:

Now we multiply that by 60 to convert the answer into minutes:

Lesson: Applied Arithmetic - 14

Introducing Averages
[Page 14 of 36]

A number that represents the characteristics of a data set.


Average is found by adding up all of the numbers you have to find the mean of,
and dividing by the number of numbers. So the mean of 3, 5, 7, 3 and 5 is 23/5 =
4.6.
Video Lesson - Explanation of Average

Averages, like ratios and rates, are variations of fractions.


The Average Formula:

Average =
So whenever you have 2 pieces of information, you can calculate the third.
For example:

The average of 3, 4 and 8 is 5, because

1. In a woodshop, one box contains 21 nails, another box contains 37


nails, and a third box contains 14 nails. ?
Answer: 24

2. A basketball team scored an average of 23 points per quarter in a


certain game. What was the total number of points scored by the
team during the game?

Answer: 92

,
so Total points =23 x 4 = 92
The average age of the members in a five-person choir is 34. If the ages of four
of the members are 47, 31, 27, and 36, what is the fifth member's age?
29
32
34
37
41

Thus , so by cross-multiplying you get 170 = 141 + x.

So x = 29 and the answer is Choice (1).


What is the average (arithmetic mean) of all the even integers greater than 24
and less than 326?
144
167
175
206
212

Whenever you're dealing with evenly spaced numbers, the average is equal to
the average of the two outer numbers.

So in this case:

Lesson: Applied Arithmetic - 18

Introducing Mean, Median, Mode And


Range
[Page 18 of 36]
Mode

The mode is the number in a set of numbers which occurs the most. So the
modal value of 5, 6, 3, 4, 5, 2, 5 and 3 is 5, because there are more 5s than any
other number.
Range

The range is the largest number in a set minus the smallest number. So the
range of 5, 7, 9 and 14 is (14 - 5) = 9. The range gives you an idea of how spread
out the data is.
The Median Value

The median of a group of numbers is the number in the middle, when the
numbers are in order of magnitude. For example, if the set of numbers is 4, 1, 6,
2, 6, 7, 8, the median is 6:
1, 2, 4, 6, 6, 7, 8 (6 is the middle value when the numbers are in order) If you
have n numbers in a group, the median is the (n + 1)/2 th value. For example,
there are 7 numbers in the example above, so replace n by 7 and the median is
the (7 + 1)/2 th value = 4th value. The 4th value is 6.

Mean Mode Median Video Lesson

Average is sometimes called Mean on standardized tests. Three other closely


related statistical terms are Median, Mode and Range.

The test-maker likes to test whether students understand the difference between
Mean, Median, and Mode, so make certain to know these definitions.
Mean = Average
Median = Middle value in a set of numbers (if the set has an even number of
elements, the number midway between the two middle values)
Mode = Most frequently appearing value in a set of numbers
Range = The difference between the greatest and least values in a set of
numbers
Let's take a look at how these concepts are tested.
{24, 40, 49, 14, 24, 36, 51}
1. What is the average (arithmetic mean) of the above set of numbers?

2. What is the median of the above set of numbers?

Median = the number in the middle after the set has been rearranged.
In the set {14, 24, 24, 36, 40, 49, 51}, 36 is the median.
3. What is the mode of the above set of numbers?

Mode = the most frequently appearing value. 24 is the only number


that appears more than once, so it is the mode
4. What is the range of the above set of numbers?

Range = difference between the greatest and least values. In this


case: 51 - 14 = 37.
5. If 49 were to be removed from the above set of numbers, what would
the new median be?

New median = the number in the middle after the set has been
rearranged: 14, 24, 24, 36, 40, 51. Since there are an even number of
elements, we want the number midway between the two middle

numbers, or
A bowler bowls five games and scores 140, 120, 175, 140, and 195 on those
games. Which of the following is true of these scores?
I. The median is higher than the mode.
II. The average (arithmetic mean) is higher than the median.
III. The range is less than half the average (arithmetic mean).

None
III only
I and II
II and III
I, II, and III
None
III only
I and II
II and III
I, II, and III

For starters, rearrange the scores in ascending order: 120, 140, 140, 175, 195.
The resulting middle number, 140, is the median, and since 140 is the only
recurring number, it is also the mode. Thus Statement I is incorrect and Choices
(3) and (5) are out.
To evaluate Statement II, we're going to have to calculate the average:

So (1) and (2) are out, and (4) must be the answer.
(But just to prove that Statement III works, the range = 195 - 120 = 75, which is
less than half the average, which we know is 154.)
If S is a set of 5 positive distinct integers with an average (arithmetic mean) of 9
and a median of 7, what is the least possible value of the largest number in the
set?
12
13
14
17
18
12
13
14
17
18

The key here is to pay close attention to the wording: You're looking for the "least
possible value" of the "largest" number in a set of "5 positive, distinct integers"
with an "average of 9" and a "median of 7." So, if the median is 7, the integers
arranged from lowest to highest are __, __, 7, __, __, and if the average is 9, the
sum of the numbers is 45.
You want the least possible value for the largest number, so that means you
want to maximize all the other numbers, and since all the numbers are distinct
integers, the two smallest numbers must be 5 and 6. You now have 5, 6, 7, __,
__, so the three numbers you know add up to 18, which means that the
remaining 2 numbers should add up to 45 - 18 = 27. Thus the fourth number
must be 13, and the largest number must be 14, like so: {5, 6, 7, 13, 14}

Lesson: Applied Arithmetic - 22

Factorials
[Page 22 of 36]

Factorials are very simple things. They're just products, indicated by an


exclamation mark. For instance, "four factorial" is written as "4!" and means
1×2×3×4 = 24. In general, n! ("enn factorial") means the product of all the whole
numbers from 1 to n; that is, n! = 1×2×3×...×n.

You may see the following on a standardized test:


7!
You should not read this to mean: emphatically seven! That exclamation point, as
a mathematical symbol, represents the factorial function, and here's what it
means:

In other words:

If you see a question involving factorials on a standardized test, you generally


won't have to calculate the value of the factorial. Instead, factorial questions tend
to test your ability to factor or reduce, so you should take note of the following:
, and so on.

Let's take a look at an in-format question.


14! - 13! =
1!
13
14
13!
13 x 13!
Since 14! =14 x 13! you can factor 13! out of your original equation:
14! - 13! = 13!(14 - 1) = 13!(13) = 13 x 13!
So the correct answer is Choice (5)

Lesson: Applied Arithmetic - 24

Probability
[Page 24 of 36]

Probability studies the possible outcomes of given events together with the
outcomes' relative likelihoods and distributions. In common usage, the word
"probability" is used to mean the chance that a particular event (or set of events)
will occur expressed on a linear scale from 0 (impossibility) to 1 (certainty).

Probability is just a special type of ratio that represents the likelihood that
something will occur.
To find probabilities when all the outcomes have the same chance of occurring,
count the number of desired outcomes and divide by the number of possible
outcomes:

Probability =

Example:
What is the probability of throwing a 5 on a fair six-sided die?
There is 1 desired outcome - throwing a 5.
There are 6 possible outcomes - one for each side of the die.

So the probability is .
Let's try an in-format question.
In a colony of 1,500 lemmings, exactly 30 lemmings are variegated. If one
lemming is chosen at random from the colony, what is the probability that it is not
variegated?

There are 1,500 lemmings and only 30 of them are variegated. That means that
nearly all the lemmings are not variegated. So the probability of picking a non-
variegated lemming is very high.
The first three probabilities are small, so you can eliminate them right off. Here it
might be easier to calculate the probability that a lemming isvariegated, and
subtract that from 1 to find the probability that a lemming is not variegated. So,
the probability that a lemming is variegated is:

And that makes the probability that a lemming is not variegated

Multiple Event Probability


[Page 26 of 36]

Many probability questions involve finding the probability of a certain outcome


after multiple events take place (a coin is tossed several times, etc.). These
questions can be slightly trickier than the previous example.
To determine multiple-event probability where each individual event mustoccur a
certain way:

 Figure out the probability for each individual event.


 Multiply the individual probabilities together.

Let's see an example:


If a fair 2-sided coin is tossed three times, what is the probability of tossing heads
the first 2 times and tails the third time?
Type the answers into the fields provided, and then click
Continue.
1. What's the probability that:

Heads will come up the first time?


Answer:

Heads will come up the second time?


Answer:

Tails will come up the third time?


Answer:

2. So, the probability of tossing Heads, Heads, Tails =

1. What's the probability that:

Heads will come up the first time?

The probability of tossing heads the first time is .

Heads will come up the second time?

The probability of tossing heads the second time is .

Tails will come up the third time?

The probability of tossing tails the third time is .


2. So, the probability of tossing Heads, Heads, Tails =

The probability of tossing Heads, Heads, Tails is


A class contains five juniors and five seniors. If one member of the class is
assigned at random to present a paper on a certain subject, and another member
of the class is randomly assigned to assist him, what is the probability that both
are juniors?

Here you can determine the probability of each individual event occurring. To
determine multiple-event probability, multiply the probability that the first event
will occur by the probability that the second event will occur.
In this case, this would translate as:

So the probability = that the first student chosen is a junior.


The probability equation is the same for the second student chosen, but the
numbers change a little. Since one student has already been chosen, there are
only nine students left from which to choose. If the first student chosen was a
junior, there are only four juniors, so the probability that the second student

chosen is also a junior is .


Now multiply the individual probabilities together. The probability that both

students chosen are juniors is

Probability Of Combinations
[Page 28 of 36]

Some probability questions involve finding the probability of certain


combinations of outcomes after multiple events take place. These questions
differ from the previous example in that a specific event does not need to have a
specific outcome, but the outcomes as a collective must exhibit certain traits.
For example:
If a fair coin is thrown twice, what is the probability of throwing one head and one
tail?
To determine multiple-event probability where individual events can have
different outcomes:

 Find the total number of possible outcomes by determining the number


of possible outcomes for each individual event and multiplying these
numbers together.
 Find the number of desired outcomes by listing out the possibilities.

Now let's solve the above example.


If a fair coin is thrown twice, what is the probability of throwing one head and one
tail?
1. How many possible ways are there for the coin to come up after 2
tosses?

To calculate the number of possible ways for the coin to come up


after 2 tosses: You know there are 2 possible outcomes for each coin
toss, so after 2 coin tosses there are 2 x 2 = 4 possible outcomes
(i.e., HH, HT, TH, TT).
If a fair coin is tossed 4 times, what's the probability that at least 3 of the 4 tosses
will come up heads?

Because the individual tosses can have different outcomes, we're best off
calculating the number of possible outcomes and then coming up with the
number of desired outcomes by listing out the possibilities.
There are 2 possible outcomes for each toss, so after 4 tosses there are a total
of 2 x 2 x 2 x 2 = 16 possible outcomes.
Let's list out all the possibilities where "at least 3 of the 4 tosses" come up heads:
H, H, H, T
H, H, T, H
H, T, H, H
T, H, H, H
H, H, H, H
So there are a total of 5 possible desired outcomes. Thus, the probability that at

least 3 of the 4 tosses will come up heads is


Combinations And Permutations
[Page 31 of 36]

Some standardized test questions may ask you about acceptable arrangements
for a number of items. We call these Combination andPermutation problems.
The key to solving these problems is to determine first whether the order of the
arrangement matters.
For example:
Five runners run in a race. The runners who come in first, second, and third
place will win gold, silver, and bronze medals respectively. How many possible
outcomes for gold, silver, and bronze medal winners are there?
If the order of the arrangement matters, you are, mathematically speaking,
looking for the number of permutations. The example above is, in fact, a
permutation problem.
Let's go to the next page to find out how we handle these types of questions.

Combinations And Permutations,


continued
[Page 32 of 36]

To find the number of permutations, simply multiply the possible number of


outcomes for the first position times the possible number of outcomes for the
second position, and so on. Just remember that the group gets smaller as you go
along.
Below is the example from the previous page. Type your
answers into the Text Boxes provided, and then click
Continue.

Five runners run in a race. The runners who come in first, second, and third
place will win gold, silver, and bronze medals respectively. How many possible
outcomes for gold, silver, and bronze medal winners are there?
x x =
# who could # remaining # remaining # of possible
come in first who could who could outcomes
place come in 2nd come in 3rd for 1st, 2nd
and 3rd
place

Five runners run in a race. The runners who come in first, second, and third
place will win gold, silver, and bronze medals respectively. How many possible
outcomes for gold, silver, and bronze medal winners are there?
5 x 4 x 3 = 60
# who could # remaining # remaining # of possible
come in first who could who could outcomes
place come in 2nd come in 3rd for 1st, 2nd
and 3rd
place
Any of the 5 runners could come in first place, leaving 4 runners who could come
in second place, leaving 3 runners who could come in third place, for a total of: 5
x 4 x 3 = 60 possible outcomes for gold, silver and bronze medal winners.

Combinations And Permutations,


continued
[Page 33 of 36]

Now that we've discussed permutation, let's move onto combinations. Read the
following example:
Alex wishes to plant three different fruit trees in his front yard. He has five
different types of fruit trees he can choose from: cherry, plum, apple, peach, and
pear. How many different combinations of fruit trees are possible?
In combination questions, such as the one above, the order or arrangement of
elements does not matter. You often don't need a complex formula to solve
combination questions. Most standardized test combination questions are simple
enough that you can list out the possibilities and count them.
However, there is a formula that you can use for more complex questions.
Go on to the next page to find out more about these two approaches.

Combinations And Permutations,


continued
[Page 33 of 36]

Alex wishes to plant three different fruit trees in his front yard. He has five
different types of fruit trees he can choose from: cherry, plum, apple, peach, and
pear. How many different combinations of fruit trees are possible?
Solving Combinations: the Listing Method:
To solve the example problem, label the five possible elements (in this case
trees) A, B, C, D and E. Now begin listing out the possibilities (keeping in mind
that arrangement doesn't matter). One trick is to list out the possibilities in
alphabetical order starting with ABC, and to work your way through the
possibilities: ABC, ABD, ABE, ACD and so on.
Solving Combinations: the Formula Method:
If you prefer not to use the Listing Method, the formula for solving for the number
of combinations is as follows:

n = the number in the larger group


k = the number you're combining (# in the smaller group)
Pick whichever method works for you.
Alex wishes to plant three different fruit trees in his front yard. He has five
different types of fruit trees he can choose from: cherry, plum, apple, peach, and
pear. How many different combinations of fruit trees are possible?
5
10
15
30
60
The Listing Method:
ABC ADE
ABD BCD
ABE BCE
ACD BDE
ACE CDE
Thus there are a total of 10 possible combinations.
The Formula Method:

possible combinations.
Basic Algebra
Algebra is a way of dealing with numbers and signs to solve a problem solving or
word problem question. You can convert the statement of the question in a
simple linear equation and solve it by using the basic rules of Algebra. For
example:

 Akbar has Rs. 10 in his pocket. He wants to buy a book costing Rs. 30.
How many Rupees does he need to buy the book.
 In Marathon in Multan, Yahyah completed the distance of 20 kilometers in
2 hours while his friend Shabir took 20% more time. How many minutes
Shabir was behind Yahyah.

If you are not a mathematics student, don't be afraid, Most of the questions on
the test involve only linear equation of one variable. Which is easy to solve in
seconds. Only a very few questions are based on two variable algebraic
equation.

Lesson: Basic Algebra - 01

Welcome to CAT Problem Solving


Algebra
[Page 1 of 21]

Algebra is a major branch of mathematics which studies relations and operations.

In this module you will review …

 Variables and equations


 Inequalities
 Converting English into algebra

As a topic, "algebra" is pretty broad. But if you've been away from formal math for
a while, the scope of this module will be comfortingly narrow. We're going to
focus on fundamentals, and we'll take our time going through them. Keep in mind
that fundamentals will be good for many points on your exam. Moreover, they'll
lay the groundwork for the more complex concepts that will help you rack up
even more points on test day. Take it one step at a time - no need to be fearful.
Algebra is precise, and non-threatening.
Let's get started!

Lesson: Basic Algebra - 02

Introducing Variables
[Page 2 of 21]

Algebra is a big word for a straightforward idea: it's a system of working with
equations in which one or more parts of the equation are unknown. These
“unknowns” are called variables and are represented in equations by letters
instead of by numbers.

A variable is a letter that acts as a placeholder for a number or a set of numbers.


For example, in the equation
6x = 18

x holds the place of 3, since (6)(3) = 18. From an equation like the one above, we
can determine the value of x and go on to use that information in a number of
ways.

Lesson: Basic Algebra - 03

Isolating Variables
[Page 3 of 21]

The textbook method to solving these equations is to isolate the variable on one
side of the equation by doing the same thing to both sides.
To isolate x in the following equation, we start by multiplying both sides by 3:
Then we add 6 to both sides:

Finally, we divide by two, and x is all alone on one side of the equation, which is
exactly what we want:

Isolate the variables in the following equations.

x =

Your equation should have looked like this:

Isolate the variables in the following equations.


x=5

a =

Your answer should have looked like this:


Multiply both sides of the equation by 6.

Divide both sides by –3.

Take the cube root of both sides.

Isolate the variables in the following equations.


x=5
a=2

y =

The easiest way to solve this equation is first to simplify the expression on the
left.

Next, you may flip the numerator and denominator on both sides (since , this
is easy).
Then solve the equation for y by isolating the variable.

Some Rules for Simplifying Equations


[Page 5 of 21]

Sometimes parentheses make equations look more complicated than they


actually are. (See how much more complicated this sentence seems?) If you
simplify the expressions on each side of the equation before solving it, your life
will be much easier.div>
You can multiply both sides of an equation or an inequality by any number
except zero.
Multiplying by zero yields a result of 0 = 0, which won’t help you to find a solution.
Avoid multiplying both sides of an equation by a variable. A variable
could be equal to zero (which would yield that useless 0 = 0 result).

If , then 2x= -6 -3 -2/3 3 6


Before we can find 2x, we must isolate the variable:

Now that we have the variable, we can use it to find other information. Here, we
had to find x so that we could figure out what 2x is:
The answer is the first choice.When questions are posed in multiple-choice
formats, multiple solution methods are possible, including back solving and
picking numbers (strategies covered elsewhere in the CAT program). Here in this
module we will always highlight the algebraic solution, because that's our topic.
Just remember that alternative approaches are often at your disposal. By the
way, did you note that this particular question asked for 2x, not just x? And did
you notice that the answer you found for x was also there among the choices?
Always read the question carefully, so you don't get caught in these sorts of
traps!

If what is the value of a? -


7 0 3 6 7
To answer this question, solve the first equation for b, then plug the result into
the second equation to solve for a: The first equation:

Then plug in 2 for b in the second equation:

The answer that fits is the fourth choice.

Introducing Inequalities
Variables can also occur in inequalities. An inequality is a statement of the
relative size of quantities, and when only straight numbers are involved,
inequalities are self-evident, e.g. 10 > 5, or 88 < 102. But when inequalities
involve variables they can become confusing.
[Page 8 of 21]

div>
Think of a variable in an inequality as representing a range of numbers.
You may use the same solution methods for inequalities as you use for equations
(with one big exception, which we'll discuss in a minute). In general, what you do
with an inequality is to isolate the variable to one side, as in:

See what we mean by "a variable in an inequality is a range"? In this


example, x can be equal to any number less than –3. In other words, any number
less than –3 will make the inequality true. For example, if xwere equal to – 4,
then:

Since the inequality is valid, we have proven that x could be equal to – 4, and of
course it could be equal to a lot of other numbers, too. But xcannot be equal to –
3, for if it were, the two sides would be equal. And xcannot be greater than –3,
because then the left side would be greater than the right.
All the rules for solving equations apply to solving inequalities as well: do the
same thing to both sides, don’t multiply by zero, and don’t multiply by a variable.

Solving Inequalities with Negatives: The


Big Exception
[Page 9 of 21]
There is one critical difference between solving an equation and solving an
inequality:
When multiplying or dividing both sides of an inequality by a negativenumber,
the inequality sign reverses.
Let’s see how this works:

which is a true statement.

But what if we hadn’t flipped the signs? Try inserting a number less than –10
for x and see what happens. Doing so should demonstrate the importance of this
rule.

Answer:
Subtract 2 from both sides, then divide by 6. No need to flip the sign since you’re
not dividing by a negative:

Answer: x < -19

Simplify the expression first:


Then solve for x :

Answer: x <11
Isolate x on one side of the equation

Flip the sign of the inequality when you divide by a negative.

If , what is one possible value for f ? -2 0 2


4 5
To isolate f on one side of the equation, do the same thing to both
sides:
Since f must be greater than 4, the last choice is our answer. Let’s try one more.

Column
Column A
B

-
Z-2
10

Column A is greater.

Column B is greater.

The columns are equal.

The relationship cannot be determined.


Solve the equation:

Don’t forget to reverse the inequality when multiplying or dividing both sides of an
inequality by a negative number. Since z is always less than –8, z – 2 will always
be less than –10, and thus Column B is greater, and that's your answer. There’s
a trap here: don’t forget that this is an inequality! The variable z will never
actually be –8, but will always be less than –8, even if only by a hairsbreadth.
Therefore, –10 is always the greater quantity.

Introducing Translation
[Page 13 of 21]

Even before you begin an algebra problem, you have to be sure that you
understand what the problem is asking. This is relatively easy when the problem
is written in math:

But what about when the problem is written in English?

What number is 16 more than the product of itself and six?


Standardized tests will often ask you to translate English words into algebraic
expressions. Let’s see how this is done.

Translating English into Math


[Page 14 of 21]

Each word problem will contain certain key phrases that can clue you in to the
algebraic functions needed for the solution. By reading carefully, you can catch
these key words and proceed to translate them into math.
Let’s look at our previous example. div>
What number is 16 more than the product of itself and six?

Type your translation into the field provided and then click
Continue.
What number is . . .
Translation: x=

Whenever you read “what” in a word problem, that’s your cue to stick in a
variable. Call that unknown number x or whatever you feel like. And whenever
you read “is” , that translates as =. So you should have typed something like: x =
It’s a good idea to avoid the letters “o” and “i” when picking letters to
stand in as variables—they’re too easily confused with the numbers “0”
and “1”.
What number is 16 more than the product of itself and six?

How would you translate this piece from English into math?
Type your translation into the field provided and then click
Continue.
What number is 16 more than . . .
Translation: x = 16 + …16 more than… means we’ll have to add 16 to
whatever comes next. So let’s write that in as: x = 16 +
What number is 16 more than the product of itself and six?
Translation:

…the product of itself and six: That’s the number in the question multiplied by six.
We called the missing number x. So, the quantity described here is 6x.
So your resulting equation should look something like:

Now it’s in a form you can solve.

Translate first, then solve.

Translation Grid
[Page 17 of 21]

This chart lists other English words you may have to translate into math. div>

Operation Verbal Cues Examples

7 more than a number


sum, plus, and, added to, more than, is 44
Addition increased by, combined with, and,
exceeds, total, greater than n + 7 = 44
difference between, minus, subtracted Tea costs 25 cents less
Subtraction from, decreased by, diminished by, less than coffee.
than, reduced by t=c - 25

One-third of a certain
of, product, times, multiplied by, twice, number is 5.
Multiplication
double, triple, half

The quotient of 7 and a


quotient, divided by, per, out of, ratio of
Division number.
_ to _

The result of 9 times y


equals, is, was, will be, the result is,
Equals is 8.
has, adds up to, costs, is the same as
9y=8

A number q is a “pequod” number if the sum of q and half of q is


a multiple of 6. Which of the following is a “pequod” number?
-1 3 4 6 10

Translate the definition of a “pequod” number into English: The sum

of qand one half q : So a multiple of six. Since all of the answer


choices are single digits, try setting the equation equal to a multiple of six that
also has one digit (like, say, 6):

So 4 is a “pequod” number, and the third choice is the answer. Alternately, you
could also pick answer choices and plug them into the equation, but you’d still
have to translate the English into algebra first. div>
The product of a and b is nine less than the number b. The sum of a and
twice b is 35. Which two equations could be used to determine the value
of a and b ?
ab = b - 9
2ab = 35
a+b-9=b
2ab - b = 35
ab - 9 = b
a + 2b = 35
ab = b - 9
a + 2b = 35
ab - 9 = b
2ab = 35
ab = b - 9
2ab = 35
a+b-9=b
2ab - b = 35
ab - 9 = b
a + 2b = 35
ab = b - 9
a + 2b = 35
ab - 9 = b
2ab = 35

Translate piece by piece:


“The product of a and b ” indicates multiplication: (a)(b)
orab “nine less than the number b” indicates subtraction: b –
9 “The product of a and b is nine less than the number b.”: ab= b –
9 (There’s one equation). “The sum of a”:
This indicates addition: a + “and twice b”: That’s 2b “The
sum of a and twice b is 35.”: a + 2b = 35 (There’s the
second equation). The fourth choice matches what we found. Note
that we don’t care whata and b actually are; the question only asks us to
translate, so that’s all we need to do.
Twenty percent of Country Y’s yearly exports come from fruit exports. One-sixth
of all fruit exports from Country Y are orange exports. If Country Y generates
$4.25 million from it’s orange exports, how much money does it generate from it’s
yearly total of exports? $21.25 million $25.5 million $106.25
million $127.5 million $153 million
Set up the problem by translating it into math: Twenty percent of Country Y’s
yearly exports come from fruits. Let x equal Country Y’s yearly export value,
and f equal the value of its fruit exports:
f = 20 percent of x
f = .2x
One-sixth of all fruit exports from Country Y are oranges. Let y equal the orange

export value: Now that we have our equations, plug in the value
given for y (the oranges) and solve:

y = 4.25 million
Now plug the value for f back into the first equation:
f = .2x

25.5 mil = .2x

So the fourth choice is correct.

Advanced Algebra
Advanced Algebra on the aptitude tests does not mean more than elementary
level. Some questions involve quadratic equation. Otherwise; the questions are
based on algebrain fractions, Order of operation precedence, and simultaneous
equations of two or three variables.
How to Deal

For dealing of question follow the rules and tricks discussed in the module. Keep
in mind, the module is limited to the horizones of normal aptitude tests. You
should not expect what is beyond the scope of the test.

Combinations and Polinomials

This module also deal with comination problems and simple euations of multiple
variables.

Advanced Algebra
[Page 1 of 23]
In this module you will review more advanced Algebra concepts, such as…

 Quadratic Equations and the FOIL Method


 Multiple Variable Systems

At some point during most standardized tests, you the test taker will confront
problems involving advanced algebra concepts like the ones in today's
module. Such concepts may have only marginal impact on your score (since
the lion's share of points usually stem from more basic concepts), but the
tougher problems tend to be multi-step and time consuming, and hence
confidence-destroying. Reviewing these more complicated concepts now will
be a smart first step toward having the knowledge, technique, and
confidence needed for a fair shake at the very toughest problems.

What is a Quadratic Equation?


[Page 2 of 23]

A quadratic equation is a univariate polynomial equation of the second degree. A


general quadratic equation can be written in the form:
ax2 + bx + c = 0

Even though you probably haven’t seen these for some time, quadratic
equations are very popular with standardized tests. Fortunately, there are a
few key rules which make quadratics easier to handle, so you can ace the
math section on test day.
The very first thing to remember is its definition: a quadratic is any equation
that can be written , where a, b, and c are numbers and x is a
variable.

is a quadratic equation, as is (in this case, b = 0 so the


second term drops out).

A quadratic is any equation that can be written in the form

Introduction to FOIL
[Page 3 of 23]

The FOIL Method is a process used in algebra to multiply two binomials. The
lesson on the Distributive Property, explained how to multiply a monomial or a
single term such as 7 by a binomial such as (4 + 9x). 7 (4 + 9x)
But, what if there was a binomial instead of a single term outside of the
parentheses? That is, what if a binomial was being multiplied by another
binomial? An example of this is given below.
(3 + 7x)(6 + 2x)
On standardized tests, quadratics usually result from multiplying two
binomials. Binomials are mathematical expressions involving the sum or
difference of a variable and a number (“x + 1” is a binomial).

In other words, an expression like can be written as :

On test day, you may have to turn an expression that’s written in one form
into its other expression. Does FOIL ring a bell?
FOIL stands for:
The product of the F irst terms, added to
…the product of the O utside terms, added to
…the product of the I nside terms, added to
…the product of the L ast terms.
Let’s take a look.
Applying FOIL
[Page 4 of 23]

div>

Let’s apply FOIL to expand an expression into a quadratic.

To expand :

First terms first: multiply x by x to get <="" p="" border="0"


align="absMiddle" height="20" width="20" class="img-responsive"
style="box-sizing: border-box; border: 0px; vertical-align: middle; display:
block; max-width: 100%; height: auto;">

O utside terms next:

Applying FOIL, continued


[Page 5 of 23]

Inside terms next:

Last terms last: (3)(–2) = –6

Combine like terms and we find:

And that’s how FOIL expands two binomials into a quadratic.


Use FOIL to determine the larger of the two quantities shown. Click on the
larger and then click Continue.

expands to
expands to

They are the same except that this second one has 16 added to it where the first
has 7. Thus the second one, , is 9 more than

expands to (the positive x and the negative x cancel each other


out). Regardless of what x is, will always be greater than .

expands to (or ). expands to


(or ). Eliminating like terms from both questions
produces -6 and -15. Since -6 is greater, is the greater quantity.

Factoring Polynomials
[Page 7 of 23]

Just as you can use FOIL to make a quadratic from two binomals, you can
get binomals from quadratics using by reversing the process.

For example: let’s turn into the product of two binomals.


Start with First Terms: Start building your parentheses:
(x )(x )

What Last terms will produce –16?


What Outer terms added to Inner terms will produce 6x?

Type your answer into the box provided then click Continue.
What factors of –16 add up to positive 6?
Answer: -2 and 8

Countinue
List out all the factors of the last term:
(1)(–16)
(2)(–8)
(4)(–4)
(–2)(8)
(–1)(16)

FOIL Example Completed


[Page 9 of 23]

Once you’ve got the last two terms you can finish the quadratic:

And that’s how you convert a quadratic into two binomials.

Factoring Out a Common Factor


[Page 10 of 23]

A factor common to all parts of a polynomial can be factored out. If you


notice that all terms in a polynomial can be divided by the same number or
variable, you can pull that number or variable out of the equation.

For example, in the equation, , the number 4 divides evenly


into each of the terms. Thus, the 4 can be pulled out so that equation reads:
.

Another example:

Since each of the terms is divisible by 3w, it can be pulled out of the
expression.
Understanding how to pull out common factors will help you clean up
equations on test day.
Column A Column B
x <0
(x + 4)(x +5) (x + 2)(x + 10)

Column A is greater
Column B is greater
The columns are equal
The relationship cannot be determined

Expanding Column A's binomial yields .

Expanding Column B's binomial yields .


Dropping like terms in both expressions, we find that Column A contains 9x, and
Column B, 12x. Since x < 0, Column A is always greater than Column B.

You can double-check your choice by picking numbers like –1, –2, and

For which of the following values of m is the above expression positive?


1
0
-1
-3
-5

There are several ways to solve this problem, but the quickest is to use reverse
FOIL to see that:

Now we can substitute values for m into a relatively simple expression. Let's start
with the first choice since it's an easy number to substitute.
Remember, zero is neither positive nor negative, so a result of zero doesn't give
us a positive number. Try the next choice.

Not that one, either. What about the next one?

Nope. The fourth choice?

It must be the last choice.

That’s positive, so -5 is our answer.

Introduction to Multiple Variable


Systems
[Page 13 of 23]

Solving for a variable isn’t too hard when there’s just one variable:

But this isn’t always the case:


Standardized tests will sometimes require you to solve equations with two
variables, but don’t worry.
They’ll always give you just what you need to do this: a second equation.

Combination
[Page 14 of 23]

When you have two variables and two equations, there are two ways to solve the
problem. The first method we’ll discuss is the combination method.
If you have two variables in two equations (or as it’s sometimes called, a
system of equations), you can solve for one variable by combining the
equations.
When you combine two equations, line up the equations and add the two
together or subtract one from the other.
Let’s look at how combination works.
If you saw this:

You could add the equations together, resulting in:

Combination, continued
[Page 15 of 23]
However, most of the time, the two equations won’t line up as cleanly as they
did in the previous example.
Sometimes it’s necessary to multiply an equation by a number in order to get
a variable to cancel out.
For example, if you add the equations 3x + 4y = 12 and x + 2y = 10 together,
neither x nor y will cancel out. But if you multiply both sides of the second
equation by –2, you’ll get somewhere:

And then adding the new second equation to the original first equation:

The y drops out. Remember, you can multiply both sides of any equation by
any number other than zero, and the equation will still be true. This is very
helpful in combination problems.

Multiplying Equations in Combination


[Page 16 of 23]

div>
Occasionally, if you want to use equation combination to get rid of a variable,
you’ll have to multiply each equation by a different number.
For example, in the following question, a good strategy for getting rid of y is
to multiply the first equation by –5 and the second equation by what number?

In the space provided, type the number you would multiply the
second equation by to make y cancel out, then click Continue.
Answer: 2

If you choose –5 for the first equation, you’ll want to select 2 for the second
equation to make the y-variable disappear:

Added together, y will drop out, and you’ll be able to solve for x.
Now let’s move on to our second method…substitution.

Substitution
[Page 17 of 23]

The second approach to solving systems of equations is the substitution


method. In this approach you use one equation to find out about one
variable, and use that information to solve the second equation.
For example, if we know that:

and:

then we can use the second equation to find the solution to the first equation.
The first step is to isolate x in the second equation:

This is known as finding x in terms of y.


Substitution, continued
[Page 18 of 23]

Now let’s replace our new definition of x in the original equation to find y.

If , then:

Don’t forget the Order of Operations!

And that’s how you substitute. If you needed to find x as well, you could solve
for y and substitute, or just plug the value for y into either equation.
You’d find that:

Substitution vs. Combination


[Page 19 of 23]

Of course, on a standardized test, you’re working under a time limit. Therefore,


it’s in your interest to recognize which method of solving systems will
be faster on any given problem.
For example, this system:
...is just begging for combination, since the y and the –y will cancel out without
any multiplication at all, and you’ll be able to quickly find the answer.

Try It For Yourself – Substitution vs.


Combination
[Page 20 of 23]

Let’s try another one. Would combination or substitution be easier with the
following system of equations?

Click either Combination or Substitution and then click


Continue.

Combination Substitution

The second equation allows you to find j in terms of k very easily. You’ll get a

fairly simple , which will make things cancel beautifully in the first
equation. Substitution works much quicker than combination here. If you
combined, you’d have to make fractions and integers cancel out — too much
room for error to make that approach a good strategy.
Column
Column A
B

b+c 0
Column A is greater
Column B is greater
The columns are equal
The relationship cannot be determined

This is a good combination problem if you look for a shortcut.


Start by putting both equations in similar forms:

Now, combine the two equations:

Adding them together, we find an exact amount for b+c. You could try multiplying
the two equations to make one of the variables drop out, but notice that all you
need to know for this problem is the quantityb+c — which we already have. b+c =
–1 which is less than 0, so the second choice, Column B, is the greater amount
and the correct answer.

A poster shop sold a total of fifty prints by Picasso and Kandinsky for a total
of $655. Kandinsky prints cost $12.50 and Picasso prints cost $15. How
much did the poster shop make from the Picasso prints?
$150
$180
$215
$475
$570

The challenge in this question is to translate the English into algebra when the
information is mixed up. But a little logic will clear things up.
Let k = the number of Kandinsky prints sold and p = the number of Picasso prints
sold. We’re told that fifty of these are sold altogether — that indicates addition,
so:
p + k = 50

What’s more, we know how much each print costs, and the total price of all of the
prints sold. So, since the (price of one Kandinsky) times (number of Kandinsky
prints) plus (price of one Picasso) times (number of Picasso prints sold) must
equal (total price of all prints sold), we can create a second equation:
15p + 12.5k = 655

You will be expected to make reasonable assumptions like this on the test.
Since the first equation is pretty simple, let’s use substitution:
p + k = 50
k = 50 – p

Substituting into the second equation:


15p + 12.5k = 655
15p + 12.5(50 – p) = 655
15p + 625 – 12.5p = 655
625 + 2.5p = 655
2.5p = 30
p = 12

So 12 Picasso prints were sold. Multiply the number of prints by the cost of a
single print to get the final answer:
(12)(15) = $180

Thus, the second choice is the answer.


Strategies to Solve Geometry Questions
with high scoring Tricks Download
MCQs
Geometry is fairly unique among the Quantitative topics. There are many more
rules you must memorize (area, perimeter, triangle ratios, etc.), and there's often
a diagram accompanying the question. That doesn't mean you can treat
geometry as completely separate from the rest of the test, but there are a few
issues to focus on.

High Scoring - Memorize the Rules

Not all geometry formula you have learnt in schools are to be recalled and
memorize. Only a few are used in the test. However; if you know those formula
or geometrical relations, you can perform well on the test. You have to know the
30:60:90 triangle ratio. You'll be tested on the area of a circle.

Geometry
The term geometry is derived from the two Greek words geo and metron. It
means “to measure the Earth.” The great irony is that the most basic building
block in geometry, the point, has no measurement at all. But you must accept
that a point exists in order to have lines and planes, because lines and planes
are made up of an infinite number of points. Geometry is the study of figures in
space. As you study geometry, you will measure these figures and determine
how they relate to each other and the space they are in. To work with geometry
you must understand the difference between representations on the page and
the figures they symbolize.

Geometry Needs Time

Success in geometry requires effort. Make a commitment to improve your


geometry skills. Work for understanding. Why you do a math operation is as
important as how you do it. If you truly want to be successful, make a
commitment to spend the time you need to do a good job. You can do it! When
you achieve success in learning geometry concepts, you will have laid a solid
foundation for future challenges and successes.
What you see is not always what is there. In space, lines define a square; on the
page, four distinct black marks define a square. What is the difference? On the
page, lines are visible. In space, lines are invisible because lines do not occupy
space, in and of themselves. Tests focus on the practical application of geometry
towards solving planar (two-dimensional) spatial puzzles.

Finding Degree Measurements


[Page 1 of 30]

See if you can find the degree measurements for ∠ ABD, ∠ ACD, and ∠ ADB.
Type your answers into the text boxes, and then click Continue.

∠ ABD =110
∠ ADB = 25
Now consider ∠ ABD. The interior angles of this triangle must also sum to 180°.
We found that ∠ ABD = 110°, so 110° + ∠ ADB + 45° = 180°, or ∠ ADB = 25°.

Angles
[Page 2 of 30]

Angles are formed when two lines or line segments intersect. The concept of
angle is one of the most important concepts in geometry. The concepts of
equality, sums, and differences of angles are important and used throughout
geometry.
The sum of angles around a point is 360°. The sum of angles along a straight line
is 180°.
Angles that lie along a straight line are said to be supplementary to each other.
Therefore, supplementary angles sum to 180° .

A good understanding of these concepts helps us to find the degree


measurements of unknown angles.

Finding Degree Measurements


[Page 3 of 30]

See if you can find degree measurements of ∠ FAB, ∠ BAC, and ∠DAE. Type
your answers into the text boxes and then click Continue.

Hint
∠ FAB = 130

Countinue
∠ FAB and the angle measuring 50° are supplementary (they sum to 180°).
Therefore, ∠ FAB must equal 130°.

Countinue

Finding Degree Measurements


[Page 3 of 30]

div>

See if you can find degree measurements of ∠ FAB, ∠ BAC, and ∠DAE. Type
your answers into the text boxes and then click Continue.

Hint
∠ FAB
∠ BAC = 50

Countinue
∠ FAB and ∠ BAC are supplementary. We know that ∠ FAB is 130°, so
∠ BAC must equal 50°.

Countinue

Finding Degree Measurements


[Page 3 of 30]
See if you can find degree measurements of ∠ FAB, ∠ BAC, and ∠DAE. Type
your answers into the text boxes and then click Continue.

Hint
∠ FAB = 130
∠ BAC = 50
∠ DAE = 30

Countinue
∠ BAC, ∠ DAE, and the angle marked 100°are supplementary. ∠ BACequals 50°,
so ∠ DAE must equal 30°.

Vertical Angles
[Page 4 of 30]

When lines intersect they form opposite (or vertical) angles. In the figure below,
there are two pairs of opposite angles: ∠ AEB and ∠ DEC, and∠DEA and ∠ CEB.
Notice that in the figure below the angles across from each other are each
supplementary to the same angle. For example, ∠ AEB and ∠BECare
supplementary, and ∠ AEB's opposite angle, ∠ DEC, is also supplementary to
∠ BEC. This leads to the following conclusion.

All opposite, or vertical, angles are equal.


∠ DEA and ∠ CEB are each supplementary to ∠ AEB. So each of these angles
equals 180° – ∠ AEB.
∠ AEB and ∠ DEC are each supplementary to ∠ DEA. So each of these angles
equals 180° – ∠ AEB.
Therefore, ∠ DEA = ∠ CEB and ∠ AEB = ∠ DEC.

Finding Degree Measurements


[Page 5 of 30]

What are the degree measurements of ∠ AED, ∠ DEC, and ∠ CEB? Type your
answers into the text boxes, and then click Continue.

Hint
∠ AED =160

Countinue
Since ∠ AED is supplementary to the angle marked 20°, ∠ AED is 160°.
Countinue

Finding Degree Measurements


[Page 5 of 30]

What are the degree measurements of ∠ AED, ∠ DEC, and ∠ CEB? Type your
answers into the text boxes, and then click Continue.

Hint
∠ AED =160
∠ DEC = 20

Countinue
Since ∠ DEC is supplementary to ∠ AED, ∠ DEC must be 20°.

Countinue

Finding Degree Measurements


[Page 5 of 30]

What are the degree measurements of ∠ AED, ∠ DEC, and ∠ CEB? Type your
answers into the text boxes, and then click Continue.
Hint
∠ AED =160
∠ DEC = 20
∠ CEB = 160

Countinue
∠ CEB is supplementary to ∠ DEC, ∠ CEB must equal 160°.

Countinue

Finding Degree Measurements


[Page 5 of 30]

What are the degree measurements of ∠AED, ∠ DEC, and ∠ CEB? Type your
answers into the text boxes, and then click Continue.

Hint
∠ AED =160
∠ DEC = 20
∠ CEB = 160
As we can see, the opposite angles are equal.

Terms And Rules


[Page 6 of 30]

Let's review some terms and rules that you might see on your standardized test.
An angle or line that is bisected has been cut in half.
A right angle has a degree measurement of 90°. It is formed whenever two lines
are perpendicular. The symbol for perpendicular lines is ⊥ .
Lines are parallel if they lie in the same plane and never intersect, no matter how
far they are extended.
If two lines are parallel, and one of the two is perpendicular to a third line, then
both lines are perpendicular to the third line.
A straight line that intersects two parallel lines is known as a transversal. Since
a transversal intersects both parallel lines at the same angle, the angles formed
mirror each other.

In the figure above, a° = e°, d° = h°, b° = f°, and c° = g°.


Back Next

Finding Degree Measurements


[Page 7 of 30]
If EF is parallel to GH in the diagram below, what are the degree measurements
of ∠ EBC, ∠ ABF, ∠ FBC, and ∠ABE? Type your answers into the text boxes,
and then click Continue.

∠ EBC = 130

Countinue
Since EF and GH are parallel, the angles formed by the transversal ADare
equivalent. ∠ EBC will have the same degree measurement as its corresponding
angle GCD, which is given as 130°.

Finding Degree Measurements


[Page 7 of 30]

div>

If EF is parallel to GH in the diagram below, what are the degree measurements


of ∠ EBC, ∠ ABF, ∠ FBC, and ∠ ABE? Type your answers into the text boxes,
and then click Continue.
∠ EBC = 130
∠ ABF = 130
∠ FBC = 50

Countinue
Since ∠ ABF is supplementary to ∠ FBC, ∠ FBC must be 50°.

Triangles
[Page 8 of 30]

A triangle has three sides and is made of straight lines. A triangle may be
classified by how many of its sides are of equal length. Or, it may be classified by
what kind of angles it has. The three angles always add to 180 degrees.
Test-makers love triangles. Not only do standardized exams always test
triangles, but they test triangles more often than they test any other geometrical
shape. The popularity of this shape means that a solid understanding of a few
basic concepts can go a long way on test day.
There are a few special triangles that you'll see again and again on your
standardized exam, but first let's begin with the basics :
A triangle has three interior angles that always sum to 180°.
Finding Degree Measurements
[Page 9 of 30]

The shortest side is always opposite the smallest interior angle and The longest
side is always opposite the largest interior angle. The sum of interior angles of a
triangle is 180 degrees.
div>

See if you can find the degree measurements for ∠ ABD, ∠ ACD, and ∠ ADB.
Type your answers into the text boxes, and then click Continue.

Hint

∠ ABD =

∠ ABD =110
∠ ADB = 25
∠ ACD = 140
Countinue
∠ ACD is supplementary to a 40° angle, so it equals 140°

Triangles: Lengths of Sides


[Page 10 of 30]

In an equilateral triangle, all three sides are the same length. In an isosceles
triangle, two sides are the same length. An isosceles triangle may be right,
obtuse, or acute. In a scalene triangle, none of the sides are the same length. A
scalene triangle may be right, obtuse, or acute.
Any side of a triangle cannot be shorter than the difference of the other two
sides.
This means that if you subtract the length of any one side from another side, the
third side will have to be at least as long as this difference. You can see this
yourself by drawing some triangles. Try to make this rule not true — you can't do
it!
Any side of a triangle cannot be longer than the sum of the other two sides.
If you add any two sides together, the third side will have to be less than that
sum.

Finding The Side Length Of A Triangle


[Page 11 of 30]

See if you can find the length of the side labeled x in the diagram below. Select
an answer and then click Continue.
3

17

19

22

Countinue
The length of side x must be greater than the difference of the other two sides
and less than the sum of the other two sides.
We know that x is greater than 12 - 7 and that x is less than 12 + 7. We can write
this as 12 - 7 < x < 12 + 7, or 5 < x < 19. Only Choice (3), 17, meets this
requirement, so it is the correct answer.

Triangles: The Side-Angle Relationship


[Page 12 of 30]

In a triangle the shortest side is always opposite the smallest interior angle and
the longest side is always opposite the largest interior angle. Every equilateral
triangle is equiangular and every equiangular triangle is equilateral. In a scalene
triangle, all the sides have different lengths and all the interior angles have
different measures.
The length of a side of a triangle is proportionate to the degree measurement of
its opposite angle.
In Δ ABC below, we know that BC > AC > AB.
How do we know this? Well, ∠ A is the largest angle, so the side opposite it, BC,
must be the longest side. ∠ B is the next largest, so the side opposite it, AC,
must be the longest. And ∠ C is smallest, so ABmust be the shortest side.
Triangles in which two of the angles (and therefore two of the sides) are of equal
length are known as isosceles triangles.

In isosceles Δ ABC in the diagram below, AB = BC.

A triangle in which all three angles (and therefore all three sides) are of equal
measure is known as an equilateral triangle. In an equilateral triangle, all three
interior angles equal 60°.

Finding The Side Length Of A Right


Triangle
[Page 13 of 30]

A triangle where one of its interior angles is a right angle (90 degrees). Famous
Pythagoras Theorem defines the relationship between the three sides of a right
triangle: C2 = A2 + B2
In a right triangle, the largest angle is always the right, or 90° angle. Therefore,
the longest side is the side across from the right angle. This side is
the hypotenuse. The other two sides are legs.
The Pythagorean theorem tells us that the square of one leg, plus the square of
the other leg, is equal to the square of the hypotenuse.

The Pythagorean theorem: a 2 + b2 = c2

Finding The Leg Length


[Page 14 of 30]

If AB and CD bisect each other at point E, what is the length of AC? Type your
answer into the text box, and then click Continue.
AB ⊥ CD
AB = 6 CD = 8
AC = 5

Countinue
Since AB and CD are perpendicular, the angles around point E are all right
angles. This means that we can draw a right triangle with legs of AEand CE, and
a hypotenuse of AC.

Since AB and CD bisect each other, , and

We can now plug the leg lengths into our Pythagorean theorem to find that:

Popular Pythagorean Ratios


[Page 15 of 30]

There are two Pythagorean theorem ratios that test makers use again and again.
One is the ratio you saw on the previous problem, in which the legs had lengths
of 3 and 4, and the hypotenuse had length 5. These integers form the most
common ratio; you'll see it all the time! Any time that you see a right triangle with
two of the three components, you'll know the other one without having to
calculate it.
Because this relationship is a ratio, it also works if we increase the size of each
component proportionately. Be on the lookout for triangles with sides that are
multiples of 3:4:5, like those with sides 6:8:10 or 9:12:15.
The other common ratio is 5:12:13, and you may see multiples of it as well.
Learn to recognize the 3:4:5 and 5:12:13 ratios in order to find lengths for these
special triangles instantly, without calculations.
Triangles: Area
[Page 16 of 30]

To find the area of a triangle, multiply the base by the height, and then divide by
2. The division by 2 comes from the fact that a parallelogram can be divided into
2 triangles. The base and height of a triangle must be perpendicular to each
other.

The area of a triangle can always be found by using the formula

Choose one side to be the bottom of the triangle: this is the base. The height is
the perpendicular distance from the uppermost angle to the base line, where the
base line is the line of the base extended infinitely in both directions.

Finding The Area Of A Triangle


[Page 17 of 30]

What is the area of Δ XYZ? Type your answer into the text box, and then click
Continue.

area = 15

Countinue
Notice that the question isn't asking for the area of the larger triangle—it's asking
for the area of the triangle designated by points XYZ. We need to find the base
and the height for Δ XYZ to plug into our formula.
This is an example of how geometry questions combine concepts. The sides of
Δ XWY are in the 5:12:13 Pythagorean ratio, so we know that WY has length 12.
But, that's not the base for our triangle! Our triangle's base is ZY, notWY.
Fortunately, we're also told that WZ is half of WY. Since WZ is 6, or half
of WY, YZ must also be 6.
We know that the height is 5, because leg XW drops perpendicularly from the
uppermost angle to the base line.

We can now use our formula to find the area:

Triangle: Perimeter
[Page 18 of 30]

The perimeters of the two triangles are in the same ratio as the sides. Perimeter
is the total distance around the outside of a triangle.
The perimeter of a triangle is the sum of its sides.

The perimeter of this triangle is 7 + 10 + 4.5 = 21.5

Finding The Perimeter Of A Triangle


[Page 19 of 30]
What is the perimeter of the isosceles triangle in the diagram below? Choose an
answer, and then click Continue.

4√2

6√2

12

8 + 4 √2

14

Countinue
The largest angle in a right triangle is always the right angle. This makes sense if
we remember that the sum of the other two angles is 180° - 90° = 90°. This is an
isosceles triangle, the other two angles must be equal, and each must be 45°.
Since sides are proportional to the angles that are opposite them, then the two
legs of the triangle must be of equal length. We know that one leg is 4, so the
other leg must also be 4.
Plug these numbers into the Pythagorean theorem to find the length of the
hypotenuse:
Then add the sides to find the perimeter:
4 + 4 + 4√2 = 8 + 4√2

The 45°/45°/90° Isosceles Triangle


[Page 20 of 30]

Triangles with interior degree measurements of 45°/45°/90° appear regularly on


standardized tests. Sometimes they are presented as part of a problem. At other
times, you will encounter isosceles right triangles as you work through a solution.
For instance, in working with a square, you may need to divide it into two
isosceles triangles. No matter where you see them, all isosceles right triangles,
such as the one we saw in the previous problem, are 45°/45°/90° triangles.
A 45°/45°/90° triangle is another special right triangle. Its sides are always in the
proportion X : X : X√2, where the hypotenuse is equivalent to X√2.

30°/60°/90° Triangles
[Page 21 of 30]

A triangle where the angles are 30°, 60°, and 90°. If you draw an altitude in an
equilateral triangle, you will form two congruent 30º- 60º- 90º triangles. Starting
with the sides of the equilateral triangle to be 2, the Pythagorean Theorem will
allow us to establish pattern relationships between the sides of a 30º- 60º- 90º
triangle. These relationships will be stated here as "short cut formulas" that will
allow us to quickly arrive at answers regarding side lengths without applying
trigonometric functions, or other means.
Another triangle that you're almost sure to see on your standardized test is the
30°/60°/90° triangle. Often, you'll see it when an equilateral triangle is divided into
two triangles of equal size. Learn the side-length ratio for the 30°/60°/90° triangle
and you'll save yourself many calculations on test day.
If we set x as the length of the shortest side (the side opposite the 30° angle),
then the side opposite the 60° angle has length x √3, and the hypotenuse has
length 2x.

Hint
Rectangles
[Page 25 of 30]

The rectangle, like the square, is one of the most commonly known
quadrilaterals. It is defined as having all four interior angles 90° (right angles).
The diagonals of a rectangle are congruent.
A rectangle is a parallelogram with:

 four right angles


 equal diagonals that bisect each other

If a quadrilateral has one of these two features, it will have the other one, and is a
rectangle. Rectangles have equal opposite sides.
The perimeter of a rectangle is the sum of the sides. Since there are two equal
lengths and two equal widths, the perimeter of a rectangle is 2l + 2w.

Interior angles of a quadrilateral must sum to 360°, so we know that ∠ S= 360° -


3(90°) = 90°. This means that the opposite sides of QRST are of equal length,
so QR=TS, QT=RS, and QS=RT.

Squares
[Page 26 of 30]

The diagonals of a square bisect one another and are perpendicular. In addition,
they bisect each pair of opposite angles.
A square is a rectangle with 4 equal sides.
QR = RS = ST = TQ
Because a square is a type of rectangle, it shares the characteristics of other
rectangles. It has four right angles, and equal diagonals that bisect each other.

Areas Of Quadrilaterals
[Page 27 of 30]

All five formulas on the list above are useful to know, but you can really get by
with only three formulas. First, you don’t need a separate formula for the
rectangle because a rectangle is a parallelogram, and thus you can use the
parallelogram formula for a rectangle. (The rectangle formula is on the list,
however, because it’s such a basic one — which you’ve known since at least
middle school — and because it works a bit differently than the parallelogram
formula.) And second, because the square is a special type of four quadrilaterals
— a parallelogram, a rectangle, a kite, and a rhombus — you don’t really need a
special area formula for a square.

The area of a parallelogram is the length of its base times the length of its height.
We find the height the same way that we found the height for triangles. The
height is the distance from the uppermost angle to the base line.

For parallelogram WXYZ, the length of its base is 5+15=20.


By dropping a perpendicular to the base to get the height, we have made a right
triangle. Do the lengths of one leg and the hypotenuse look familiar? We have
one of our special triangles: the 5:12:13 Pythagorean ratio. This gives us our
height, since we know that VW must be 12.
Now we can find the area, which is 20 × 12 = 240.

Areas Of Rectangles And Squares


[Page 28 of 30]

The area of a square can be found by multiplying the base times itself. This is
similar to the area of a rectangle but the base is the same length as the height. If
a square has a base of length 6 inches its area is 6 × 6 = 36 square inches.
When we're talking about rectangles, we often call the base the length, and we
call the height the width. Since the base and height for squares are equal, we
often simply refer to them as the sides of the square.
The area of rectangle WXYZ is its length times its width, so in the example above
the area is 12 &time; 20 = 240.

The area of square ABCD is the length of its side squared, so in the example
above the area is 7 &time; 7 = 49.

Finding The Area Of Quadrilaterals


[Page 29 of 30]

See if you can apply what you've just learned to the following question. Select an
answer and then click continue.
In the diagram, the blue region is a square with an area of 25. If EH has a length
twice that of IH, and HG has a length four times that of HK, then what is the
perimeter of rectangle EFGH?
60

130

200

270

340

Countinue
The area of a square is a side times a side, or s 2. We are given the area which
we can then use to find the sides IH and HK:

Since the length of the rectangle is 4 times the side, it equals 20. Since the width
of the rectangle is 2 times the side, it equals 10. The perimeter of the rectangle is
therefore 2(20)+2(10)=60.

Intermediate Geometry
In this module of intermediate geometry, you will learn some basic figures and
concepts to a medium level. The same topics have been covered in the basic
level. If you have completed the basic module successfully, you will not feel the
topics in this intermediate module alien to you.

Definitions and Notations

The most important things on the geometry section of the test are definitions and
notations. Many of the non-math test takers feel difficulties in understanding the
question because they get confused by the definitions. For example they might
not understand what is the meaning of congruent. Obviously, they must learn the
definitions and notations used on the test first.
Topics in Module

Aptitude Test

You are going to take an aptitude test not an achievement test, the test asks
nothing about the appropriateness of accounting reserves or the correct discount
rate to use in financial models. If it did, undergraduate business and economics
majors would have a distinct advantage. Asking questions about geometry levels
the playing field.
The best news in this regard is that the handful of geometry questions you will
encounter on the test can generally be solved using only a small set of geometric
principles. These involve parallel lines, triangles, quadrilaterals, circles, and a few
regular cylinders. You need not to learn each and every theorem on the
geometry.

CAT's Intermediate Geometry Module


[Page 1 of 24]

div>

In this module we will continue our review of geometry fundamentals. In the


Geometry Basics Workshop, we learned about lines, angles, and triangles.
Today we will discuss some other basic shapes you are likely to see on your
exam:
Quadrilaterals:
Quadrilaterals include parallelograms, rectangles and squares. In this section, we
will learn about angles and degree measures in quadrilaterals, and we will
practice finding perimeters and areas of these basic shapes.
Circles:
This section will provide a review of circle elements and formulas, and will allow
you to apply what you've learned to sample problems.
Let's begin.

Quadrilaterals
[Page 2 of 24]
A quadrilateral is a four-sided closed figure with straight sides.Parallelograms,
rectangles, and squares are all quadrilaterals.
The interior angles of a quadrilateral always sum to 360°.

w° + x° + y° + z° = 360°
Most of the quadrilaterals that you'll see on your standardized test will either be
squares or rectangles. Often they will be combined with triangles and circles in
some way.

Parallelograms
[Page 3 of 24]

A parallelogram is a quadrilateral with:

 Opposite sides parallel


 Opposite angles equal

These two properties go together: if the opposite angles are equal, the opposite
sides will be parallel, and vice versa.
Adjacent angles of a parallelogram are supplementary.
ADC = ABC
Since quadrilateral ABCD has opposite angles that are equal, it is a
parallelogram. That means that AD is parallel to BC, AB is parallel toDC, and
DAB = DCB.

Parallelograms: Finding Degree


Measures
[Page 4 of 24]

Task: In parallelogram FGHI, what is the degree measurement of <IFG? Type


your answer into the text box, and then click Continue.

< IFG = 105


Since opposite angles in a parallelogram are equal, we know that < IFGhas the
same degree measurement as < IHG.
To find < IHG, we need to know the value of a. Since adjacent angles in a
parallelogram are supplementary, we know that 15a + 5a + 80 = 180. We can
solve for a:
20a = 100
a=5
< IHG and therefore < IFG, have a degree measurement of 5(5) + 80 = 105.

Parallelograms: Practice
[Page 5 of 24]

Task: Select the correct answer and then click Continue.

In the figure above, if ABCD is a parallelogram, then x =

20°

30°

40°

50°

60°

A key feature in this figure is the line that extends from the base of the
parallelogram. This line is composed of two supplementary angles; one is
marked 50°, and the other is ABC. Together, these angles must equal 180°, the
degree measure of a straight line. Therefore, ABC measures 180° – 50° =
130°. Now that we have handled this external angle and obtained this valuable
piece of information, we can deal with the angles of parallelogram ABCD.
Since ABCD is a parallelogram, AB is parallel to DC and AD is parallel toBC. In
addition, diagonal AC functions as a transversal in this figure, cutting both sets of
parallel lines at the same angle and creating sets of equivalent angles.
Therefore, we know that ACB = DAC = 20°, and BAC = DCA = x.
There are now two approaches we can use to determine x:

1. Adjacent angles of a parallelogram are supplementary:

2. The sum of angles in a triangle equals 180°:

Choice (B) is correct.

Rectangles
[Page 6 of 24]

A rectangle is a parallelogram with:

 four right angles


 equal diagonals that bisect each other

If a quadrilateral has one of these two features, it will have the other one, and is a
rectangle. Rectangles also have equal opposite sides.

The perimeter of a rectangle is the sum of the sides. Since there are two equal
lengths and two equal widths, the perimeter of a rectangle is 2l + 2w.
Interior angles of a quadrilateral must sum to 360°, so we know that < S= 360° -
3(90°) = 90°. This means that the opposite sides of QRST are of equal length,
so QR=TS, QT=RS, and QS=RT.

Squares
[Page 7 of 24]

A square is a rectangle with 4 equal sides.

QR = RS = ST = TQ
Because a square is a type of rectangle, it shares the characteristics of other
rectangles. It has four right angles, and equal diagonals that bisect each other.

QU = RU = TU = SU
Rectangles And Squares: Practice
[Page 8 of 24]

Tas: Select the correct answer and then click Continue.

If E is the intersection of two diagonals of rectangle ABCD, what is the perimeter


of triangle AEB?
24

25

34

38

40

In order to find the perimeter of AEB, we must find the lengths of AE and ED.
We are told that the endpoint E is also the intersection of the rectangle's
diagonals. Since diagonals of a rectangle bisect each other, this means
that AE must be half the length of diagonal AC, and DE must be half the length of
diagonalDB. Furthermore, diagonal AE and diagonal DE must be equal since a
rectangle's diagonals are equal in length.
To find the length of either diagonal, we can use the rectangle's length and width
as given in the diagram to construct a right triangle with a diagonal as the
hypotenuse (see diagram above). We can then use the Pythagorean theorem to
find the length of the diagonal. Let's use ADC to find AC:
Note that our result is a Pythagorean ratio (5:12:13). We calculated our result to
illustrate the use of the Pythagorean theorem, but on test day it will be to your
advantage to know and use side ratios to cut down on calculation time.
Since both AE and ED are half the length of the diagonal, together they equal the
full length of the diagonal, or 13. Added to side AD, we have a total perimeter of
13 + 12 = 25.

Areas Of Quadrilaterals
[Page 9 of 24]

The area of a parallelogram is the length of its base times the length of its height.
We find the height the same way that we find the height for triangles. The height
is the distance from the uppermost angle to the base line.

For parallelogram WXYZ, the length of its base is 5+15=20.


By dropping a perpendicular to the base to get the height, we have made a right
triangle. Do the lengths of one leg and the hypotenuse look familiar? We have
one of our special triangles: the 5:12:13 Pythagorean ratio. This gives us our
height, since we know that VW must be 12.
Now we can find the area, which is 20 × 12 = 240.
Areas Of Rectangles And Squares
[Page 10 of 24]

When we're talking about rectangles, we often call the base the length, and we
call the height the width. Since the base and height for squares are equal, we
often simply refer to them as the sides of the square.

The area of rectangle WXYZ is its length times its width, so in the example above
the area is 12 × 20 = 240.

The area of square ABCD is the length of its side squared, so in the example
above the area is 7 × 7 = 49.

Finding The Area Of Quadrilaterals


[Page 11 of 24]

div>

Task: Select the correct answer and then click Continue.


In the diagram, the blue region is a square with an area of 25. If EH has a length
twice that of IH, and HG has a length four times that of HK, then what is the
perimeter of rectangle EFGH?
60

130

200

270

340

The area of a square is a side times a side, or . We are given the area which
we can then use to find the sides IH andHK:

Since the length of the rectangle is 4 times the side, it equals 20. Since the width
of the rectangle is 2 times the side, it equals 10. The perimeter of the rectangle is
therefore 2(20)+2(10)=60.

Area Of A Quadrilateral, Continued


[Page 12 of 24]

Task: Select the correct answer and then click Continue.


If all angles in the figure above are right angles, what is the area of the figure?
34

40

42

48

52

The figure in this question is not a quadrilateral, but it is composed of a number


of quadrilaterals. A glance at the diagram will show that the figure can be broken
into three rectangles — a small one at the top, a small one of equal size at the
bottom, and another vertically-oriented rectangle in the middle. If we knew the
dimensions of each, we could find the total area easily. But we're only given
some of the dimensions, so we'll need to use what we're given to find the rest.
Let's begin with the vertical lengths:
Vertical lengths: The only vertical length we're missing is the length of the
longest vertical segment on the right side (the 'cut out' side) of the figure. Since
the figure has a total height of 12 and we're given the 2 other vertical lengths as
being 2 and 2, this length must equal 12 – (2 + 2) = 12 – 4 = 8.
Horizontal lengths: We're missing two horizontal lengths from the small
rectangle at the bottom of the figure. But, according to the diagram, the
dimensions of this rectangle correspond to the dimensions of the uppermost
rectangle. Therefore, missing lengths are 1 and 4 (see diagram above).
We can now compute the area: The top rectangle has a length of 4 and a height
of 2 for an area of 4 × 2 = 8 The middle rectangle has a length of 4 – 1 = 3 and a
width of 8 for an area of 3 × 8 = 24. The bottom rectangle has a length of 4 and a
height of 2 for an area of 4 × 2 = 8.
The total area of the figure is 8 + 24 + 8 = 40. Choice (B) is correct.

Circles
[Page 13 of 24]

The second type of geometric figure we will study today is the circle.

A circle is defined as the set of all points that are the same distance from
the center. Since the sum of the angles around any given point is 360°, a circle
measures 360° around its center.
The radius is the distance from the center to any point on the circle.
The diameter is a straight line from one point on a circle to another that passes
through the center of the circle. A circle's diameter is always twice the length of
the circle's radius.

In the diagram above, Q is the midpoint, AQ, CQ, and BQ are all radii,
and AQB is a diameter.
A circle is always named after its center. The circle above is Circle Q.

Definitions Of Tangent Line, Chord,


Circumference, And Arc
[Page 14 of 24]

div>
A straight line that is tangent to a circle could be extended infinitely in either
direction and would still touch the circle at only one point. A right angle is formed
by the tangent line and the point's corresponding radius.
The circumference is the distance around a circle.
An arc is a portion of the circumference. An arc's length can be measured in
units, or in degrees. An arc encompassing the shorter section of the
circumference between points is called a minor arc and the one encompassing
the longer section is called a major arc.
A chord is a straight line from one point on the circumference to another point on
the circumference. It may or may not pass through the center (in other words,
diameters are chords, but chords are not necessarily diameters).

An Example Of A Tangent Line, Chord,


Circumference, And Arc
[Page 15 of 24]

In the figure above, the entire line of the circle (passing through pointsA, B, E,
and C) is the circumference, YZ is a tangent line, BEC is aminor arc, BAC is
a major arc, and BDC is a chord.
Arc BEC has a measure of 100°. Since there are 360° in a circle, this arc

represents of the circumference.

Finding A Circumference
[Page 16 of 24]
Pi (symbolized by ) is the ratio of a circle's circumference to its diameter. The
value of is approximately 3.14.

To find the circumference of a circle, multiply its diameter by . Since the


diameter is twice the radius, the circumference equals 2 times the radius, or 2 r.

In the figure above, W is the center of the circle, and WX = 9. The circumference
is 2 r or 2 (9) = 18 .

Most questions will leave the symbol in the answer choices. If isn't in the
answer choices, you can approximate it as 3, or 3.14 if you need more precision.

Find The Circumference Of The Circle


[Page 17 of 24]

Task: Find the circumference of Circle N and type it into the text box, then click
Continue.

Circumference = 12
We know that minor arc JK represents of the circumference. Therefore,
the circumference must be three times the length of minor arc JK, or 3(4) = 12.
Task: Now try Circle O. Type its approximate circumference into the text box,
entering as pi. Click Continue when you're done.

Circumference =

Countinue

Find The Circumference Of The Circle,


continued
[Page 18 of 24]

Task: Now try Circle O. Type its approximate circumference into the text box,
entering as pi. Click Continue when you're done.

Circumference = 7
POQ is a diameter of Circle O since it is a straight line that connects two points
on the circumference of the circle and passes through the center. The
circumference is the diameter multiplied by . In this case, the circumference is 7

Area Of A Circle
[Page 19 of 24]

The area of a circle is multiplied by the radius squared, or .

The area of Circle M is


Task: Select the correct answer and then click Continue.

In the figure above, what is the area of Circle D?

9
18
102.5
81
270
CDE is the diameter since it passes through the center. The radius will be half of
the diameter, so the radius is equal to 9. Thus, the area of this Circle D is

Task: Select the correct answer and then click Continue.

In the figure above, P is the center of the two circles. If , what is the area
of the shaded region?

To answer this question, we must first find the area of the large circle and the
area of the small circle. Once we have these values, we can then subtract the
smaller area from the larger to obtain the area of the shaded region.
The diagram indicates that the large circle has a radius of length y. Therefore its
area is . Likewise, the small circle has a radius of length x, so its area
is , and the difference between the two areas is .

However, the question stem tells us that , and the answer choices are
expressed in terms of x alone. Re–expressing y in terms of x, we
get . Substituting 3x into the expression above, we can determine

the area of the shaded region:


Choice (C) is correct.

Arc Length
[Page 22 of 24]

The arc length is the measure of the distance along the curved line making up
the arc. It is longer than the straight line distance between its endpoints (which
would be a chord)
If you know the radius of a circle and the measure of an angle, you can find the
length of the arc that is formed by that angle. Let's see how

Let's find the length of minor arc XZ above.

The sector of Circle Y to which minor arc XZ corresponds is 60°. Since 60° is of

the circle (a circle is 360°), the length of minor arc XZ is of the circle's
circumference.

The circumference of Circle Y is . The length of minor arc XZ is of 6 ,


or .
Task: Select the correct answer and then click Continue.
In the figure above, if Circle O has a circumference of 24 , then RS + TU =

Arc RS represents , or , of the total circumference of Circle O. Since


ROS forms opposite angles with TOU, TOU must also measure 45°, and

arc TU is also equal to of the circle circumference. Therefore, the arc lengths

together equal of 24

Choice (D) is correct.

Advanced Geometry
There are a significant number of plane geometry questions on the test. Plane
geometry is crucial background for a number of other topics, such as coordinate
geometry. Thus, it is recommended that you review the following plane geometry
sections.
Plane geometry is the science of measurement. Certain assumptions are made
about undefined quantities called points, lines, and planes, and then logical
deductions about relationships between figures composed of lines, angles, and
portions of planes are made based on these assumptions. The process of
making the logical deductions is called a proof. In this summary, we are not
making any proofs but are giving the definitions frequently used in geometry and
stating relationships that are the results of proofs.

How to Solve

Most geometry problems on the Test require straightforward calculations based


on geometry concepts. However, some problems measure your insight into the
basic rules of geometry. For this type of problem, you should practice a lot of
questions to develop an insight of what the questions are?

Scale of Drawings

Only a few questions on NAT and GAT but about one-fourth of the math
problems on the SAT involve geometry. Fortunately, the figures on the SAT are
usually drawn to scale. Hence, you can check your work and in some cases even
solve a problem by “eyeballing” the drawing. On the other hand the drawings on
GMAT, GRE, GAT, and NAT are drwan not to scale so you can not figure out the
result just by viewing the question.

CAT's Geometry Challenge Module


[Page 1 of 32]

In this module we will review:

 Circles
 Inscribed and Circumscribed Figures
 Solids
 Coordinate Geometry

As your score improves you'll see more advanced geometry problems because
of the way that the CAT adapts to your performance. These more challenging
problems may include geometry concepts that are a bit more advanced, or they
may simply combine concepts to make the problem more complicated.
Managing these types of questions is important in maintaining a high score, so
reviewing these more complicated problems is a smart first step.
Let's get started!
Introduction To Circles
[Page 2 of 32]

First, let's review the terms we'll see used in circle problems.
A circle is defined as the set of all points that are the same distance from
the center.
The radius is the distance from the center to any point on the circle.
The diameter is a straight line from one point on a circle to another that passes
through the center of the circle. A circle's diameter is always twice the length of
the circle's radius.

In the diagram above, Q is the midpoint, AQ, CQ, and BQ are all radii,
and AQB is a diameter.
A circle is always named after its center. The circle above is Circle Q

Definitions Of Tangent Line, Chord,


Circumference, And Arc
[Page 3 of 32]

A straight line that is tangent to a circle could be extended infinitely in either


direction and would still touch the circle at only one point. A right angle is formed
by the tangent line and the point's corresponding radius.
The circumference is the distance around a circle.
An arc is a portion of the circumference. An arc's length can be measured in
units, or in degrees. An arc encompassing the shorter section of the
circumference between points is called a minor arc and the one encompassing
the longer section is called a major arc.
A chord is a straight line from one point on the circumference to another point on
the circumference. It may or may not pass through the center (in other words,
diameters are chords, but chords are not necessarily diameters).

An Example Of A Tangent Line, Chord,


Circumference, And Arc
[Page 4 of 32]

In the figure above, the entire line of the circle (passing through pointsA, B, E,
and C) is the circumference, YZ is a tangent line, BEC is aminor arc, BAC is
a major arc, and BDC is a chord.
Arc BEC has a measure of 100°. Since there are 360° in a circle, this arc

represents of the circumference.

Finding A Circumference
[Page 5 of 32]

Pi (symbolized by Π) is the ratio of a circle's circumference to its diameter. The


value of Π is approximately 3.14.
To find the circumference of a circle, multiply its diameter by Π. Since the
diameter is twice the radius, the circumference equals 2Π times the radius, or
2Π r.
In the figure above, W is the center of the circle, and WX = 9. The circumference
is 2Π r or 2Π(9) = 18Π.
Most questions will leave the Π symbol in the answer choices. If Π isn't in the
answer choices, you can approximate it as 3, or 3.14 if you need more precision.

Find The Circumference Of The Circle


[Page 6 of 32]

Task: Find the circumference of Circle N and type it into the text box, then click
Continue.

Circumference = 12

We know that minor arc JK represents of the circumference. Therefore,


the circumference must be three times the length of minor arc JK, or 3(4) = 12.
Find The Circumference Of The Circle,
continued
[Page 7 of 32]

Task: Now try Circle O. Type its approximate circumference into the text box,
entering Π as pi. Click Continue when you're done.

Circumference = 7Π
POQ is a diameter of Circle O since it is a straight line that connects two points
on the circumference of the circle and passes through the center. The
circumference is the diameter multiplied by Π . In this case, the circumference is
7Π.

Area Of A Circle
[Page 8 of 32]

The area of a circle is multiplied by the radius squared, or .


The area of Circle M is

Finding The Area Of The Circle


[Page 9 of 32]

Task: Choose an answer, and then click Continue.

18Π

102.5

81Π

270
CDE is the diameter since it passes through the center. The radius will be half of
the diameter, so the radius is equal to 9. Thus, the area of this Circle D is

Arc Length
[Page 10 of 32]

If you know the radius of a circle and the measure of an angle, you can find the
length of the arc that is formed by

Let's find the length of minor arc XZ above.

The sector of Circle Y to which minor arc XZ corresponds is 60°. Since 60° is of

the circle (a circle is 360°), the length of minor arc XZ is of the circle's
circumference.

The circumference of Circle Y is . The length of minor arc XZ is of 6Π,


or Π.

Inscribed Figures
[Page 11 of 32]

An inscribed figure is one that is drawn inside another, usually with the edges
touching. For example if you drew a triangle inside a square, the triangle is
inscribed in the square.
Easy and intermediate level geometry questions usually test single figures: one
triangle, one quadrilateral, or one circle. More challenging geometry questions
combine figures. Such combination figures may include overlapping
quadrilaterals or triangles paired with circles.
A figure is inscribed within a circle if all of its vertices lie on the circumference of
the circle.

In the diagram above, quadrilateral ABCD is inscribed within Circle M.

Circumscribed Figures
[Page 12 of 32]

A geometric figure that is drawn around another geometric figure so as to touch


all its vertices is called Circumscribed. There are three different cases when a
figure is circumscribed around another geometric figure:

 Case 1: When a polygon is circumscribed around a circle


 Case 2: When a circle is circumscribed around a polygon.
 Case 3: When a polygon is circumscribed around another polygon

A figure is circumscribed around a circle if each side of the figure is tangent to


the circle.
Any rectangle circumscribed around a circle must be a square.
In the diagram above, square EFGH is circumscribed around Circle N.

The Area Of Circumscribed Squares


[Page 13 of 32]

Standardized tests will often ask you to find the area of a circumscribed square.

To find the area of square IJKL, we need to know the length of a side. Let's see if
we can use the circumference of the inscribed circle to find this measurement.
We know that we can figure out the circle's diameter and radius from the
circumference. How does this help? Well, the diameter of the circle is the same
length as each side. Because squares have parallel sides, any line that is
perpendicular to IL and JK and that connects these two lines will have the same
length as IJ. The diameter of Circle O is one such line.

Circumference equals . Since Circle O has a circumference of 8 , its


diameter is 8. So, each side of the square is also 8.
Therefore, the area of IJKL is

Inscribed Triangles
[Page 14 of 32]

A circle that inscribes a triangle is a circle contained in the triangle that just
touches the sides of the triangle.
div>

If a triangle is inscribed in a circle so that one of its sides is a diameter of the


circle, then the triangle is a right triangle, and the diameter is the hypotenuse.
Otherwise, the triangle is not a right triangle.

Δ ABC is a right triangle because side AB passes through R, the center of the
circle. AC and CB are legs, and AB is the hypotenuse.
Δ DEF is not a right triangle because no side passes through center S

Finding The Diameter


[Page 15 of 32]

Task: Find the diameter of Circle O. Type your answer into the text box, and then
click Continue.
Diameter = 5
Since ABC has diameter AOC as one side, ABC is a right triangle with
hypotenuse AC.
This is a typical example of how you can be rewarded for being a savvy test-
taker. Did you recognize the common Pythagorean ratio 3:4:5 that we learned in
Geometry Basics? If you did, you realized that the hypotenuse, AC, equals 5.
Since AC is also the circle's diameter, the correct answer is 5.

Solids
[Page 16 of 32]

Solid figures are 3-dimensional or 3D figures that have width,depth and height.
They take up space or volume.
Example of solid figures: Sphere, Cube, Cylinder, Cone, Cube, Rectangular
Prism.
As you encounter more challenging questions, you will also encounter more
three-dimensional geometric shapes.
Three dimensional objects, or solids, have length, width, and depth. The most
common solids on standardized exams are rectangular boxes,cubes,
and cylinders.
A line segment that connects adjacent vertices of a solid is an edge.

You may be asked to find the volume of a solid or its surface area.
For the solids that you're most likely to see on your exam, you can find the
volume by multiplying the area of one face by the length of the dimension
perpendicular to that face. We'll look at some examples that will help make this
description clear.
Surface Area: A solid's surface area is the sum of the areas of each two-
dimensional surface.
Now let's see some examples.

Rectangular Boxes
[Page 17 of 32]

A rectangular box is a solid with six rectangular faces. All of its angles are right
angles.

The volume of a rectangular box is equal to its length times width times height.
Total surface area is the sum of the areas of each of the 6 sides. Since each side
will have the same area as the one opposite it, we can find the surface area of
each of the three visible sides, double them to account for the sides we can't see,
and then add them.
Task: If a rectangular solid has a length of 6, a width of 4, and a height of 3, what
is its total surface area? Type your answer into the text box and then click
Continue.
Surface Area =

Rectangular Boxes
[Page 17 of 32]

A rectangular box is a solid with six rectangular faces. All of its angles are right
angles.
The volume of a rectangular box is equal to its length times width times height.
Total surface area is the sum of the areas of each of the 6 sides. Since each side
will have the same area as the one opposite it, we can find the surface area of
each of the three visible sides, double them to account for the sides we can't see,
and then add them.
Task: If a rectangular solid has a length of 6, a width of 4, and a height of 3, what
is its total surface area? Type your answer into the text box and then click
Continue.
Surface Area = 108
To find the total surface area, take the area of each of the visible sides, double
them to account for the opposite sides which cannot be seen, and then add
them.
Side one is 6 × 3, side two is 4 × 3, and side three is 6 × 4, so doubling each side
and adding gives us a surface area of: 2(6)(3) + 2(4)(3) + 2(6)(4) = 108.

Cubes
[Page 18 of 32]

A cube is a three dimensional shape that features all right angles and a height,
width and depth that are all equal.
A cube is a box in which all sides are squares.
Every edge in a cube has the same length: Length = Width = Height. Therefore,
to find the volume, we need only find the length of one side, and raise it to the
third power.
Task: Find the volume of a cube with an edge of 4. Type your answer into the
text box and then click Continue.

Volume =

Cubes
[Page 18 of 32]

A cube is a box in which all sides are squares.

Every edge in a cube has the same length: Length = Width = Height. Therefore,
to find the volume, we need only find the length of
Task: Find the volume of a cube with an edge of 4. Type your answer into the
text box and then click Continue.
Volume = 64

The cube has a volume of .

What if the question had asked for the surface area of the cube? To find the area
of any one face of a cube, we use the formula for the area of a square: .
Since all 6 faces of a cube have the same area, the total surface area is

Cylinders
[Page 19 of 32]

This shape is similar to a can. The surface area is the area of the top and bottom
circles (which are the same), and the area of the rectangle (label that wraps
around the can)
A cylinder is a uniform solid with two opposing circular faces. The surface area
between the faces is the lateral area. This lateral surface area of a cylinder is
equal to the circumference of the cylinder times the distance between the faces.

To find the volume of a cylinder, find the area of one of its circular faces, and
multiply it by the distance between the faces.
Task: If a cylinder has a radius of 8, and the distance between its circular faces
is 20, what is its volume? Type in your answer and then click Continue.

Volume =

Task: If a cylinder has a radius of 8, and the distance between its circular faces
is 20, what is its volume? Volume = 1280Π
The volume is the area of one circular face times the distance between the
circular faces. The area of the face is times 8 squared, so we have

Task: If a cylinder has a radius of 8, and the distance between its circular faces
is 20, what is its volume? Type in your answer and then click Continue.

Volume = 1280
Task: If a cylinder has a radius of 2 and the distance between its circular faces is
10, what is its lateral surface area? Type in your answer and Area =
Area = 40
The lateral surface area is the circumference times the distance between the
faces, or

Coordinate Geometry: The Coordinate


Plane
[Page 20 of 32]

In coordinate geometry, points are placed on the "coordinate plane" as shown


below. It has two scales - one running across the plane called the "x axis" and
another a right angles to it called the y axis. (These can be thought of as similar
to the column and row in the paragraph above.) The point where the axes cross
is called the origin and is where both x and y are zero.

Coordinate geometry is based on a 2-dimensional plane surface. The planes are


defined by an x-axis (horizontal) and a y-axis (vertical).

The plane looks like this:


Locations On The Coordinate Plane
[Page 21 of 32]

Points on the two-dimensional coordinate plane are defined with two coordinates:
(x, y). The x-coordinate measures the horizontal distance from the y-axis.
The y-coordinate measures the vertical distance from the x-axis.
By convention, the axes cross at the coordinates (0, 0). The x-coordinate
increases to the right, and the y-coordinate increases upward.
So, a point with coordinates (5, –4) lies 5 units along the x-axis to the right of (0,
0) and 4 units along the y-axis downward, below the x-axis.
Because the axes cross at (0, 0) the coordinate plane is divided into
quadrants. x and y are both positive in the upper right quadrant, while
both x and y are negative in the lower left quadrant. In the upper left
quadrant, x is negative and y is positive, and in the lower right quadrantx is
positive and y is negative.

Distances On The Coordinate Plane


[Page 22 of 32]

When two points have either the same x-coordinate or the same y-coordinate, we
can find the distance between them by simply adding or subtracting the
coordinates that vary.
Taskl: Find the distance between the two points above, type it into the text box,
and then click Continue.
Distance =

Distance = 18
Both marked points lie 7 units to the right of the y-axis. Therefore, they lie on a
line parallel to the y-axis. This line is indicated in the above diagram by a dotted
green line. To find the distance between them, we need to find the distance of
the dotted line. The (7, 6) point is 6 units from the x-axis, and the (7, –12) point is
12 units from the y-axis. Therefore, the two points are 6 + 12 = 18 units away
from each other

Distances On The Coordinate Plane,


continued
[Page 23 of 32]

div>

What if we are asked to find the distance between two points on the coordinate
plane that do not share a coordinate? We have to use another approach.
First, we draw a right triangle, setting the distance between the two points to be
the hypotenuse.
We then use the Pythagorean theorem and our knowledge of triangles to find the
length of that hypotenuse.
Task: Let's find the distance between the points (–5, – 3) and (7, –12). We can
draw lines perpendicular to the x and y-axes, plus one directly between the
points, to create a right triangle. Now, use what you've just learned to find the
distances between the points that share coordinates. These are the legs of the
triangle. Then, use your knowledge of triangles to find the answer. Type in your
answer, and then click Continue.

Distance between (–5, –3) and (7, –12) = 15


First, find the distances that make up the legs of the triangle. The distance from
(–5, –3) to (7, –3) is 5+7=12. The distance from (7, –3) to (7, –12) is 12 – 3 = 9.
Now, we could use the Pythagorean theorem to get the hypotenuse. But, if we
recognize that we have a 3:4:5 ratio, we save ourselves the work! The legs of 9
and 12 are products of 3 times 3, and of 3 and 4, respectively, so our hypotenuse
must be the product of 3 and 5, or 15.

Finding The Di stance Between Points


[Page 24 of 32]

Task: Try the in-format question below. Use your scratch paper to sketch your
graph, and than use your knowledge of distances and triangles to help you find
an answer. Select an answer, and then click Continue .
What is the distance between points (1, 4) and (5, –9) on the coordinate plane?
13
14

15

Draw a right triangle, with the distance between the designated points as the
hypotenuse.
The legs have lengths of 5 – 1 = 4 and 4 + 9 = 13. We can plug these numbers
into the Pythagorean theorem to find:

Slope
[Page 25 of 32]

Slope is the measure of the steepness of a line. Slope is normally described by


the ratio of the "rise" divided by the "run" between two points on a line.
All lines have a slope that indicates the direction (rise or fall) and steepness of
the line.
You may have learned that slope is defined as rise over run. This is a ratio, and

we can write it as a fraction: . The change in y is the


difference between two y-coordinates on the line, and the change in x is the
difference between two x-coordinates on the line.
Lines with a positive slope move upward toward the right. Lines with a negative
slope move downward toward the right.

Steepness Of A Slope
[Page 26 of 32]

The higher the value of the slope, the greater the relative change in y(vertical
distance) compared to x (horizontal distance). Therefore, a slope of a higher
value is steeper than a slope of a lower value.

The line with a slope of 1 increases 1 unit of y for each unit of x.


The line with a slope of 2 increases 2 units of y for each unit of x, and is therefore
steeper.

Finding The Slope


[Page 27 of 32]
Task: What is the slope of a line that passes through the points (5,8) and (–2,6)?
Use the slope equation to find the answer, type it into the text box, and then click
Continue.

Slope =

Plug the numbers into the formula for slope. (Note: it doesn't matter which set of
coordinates we designate as 1 and which we designate as 2. However, we
cannot 'mix' coordinates by choosing from one set and from the other. We
must be consistent.).
Plugging in, we find:

Equations Of Lines
[Page 28 of 32]

All straight lines on two-dimensional coordinate planes can be expressed by the


equation y = mx + b, where m is the slope, b is the point at which the line
intercepts the y-axis, and x and y are the x and y-coordinates.
The slope and the y-intercept are constants. They define the line. For every point
on the line, there is one set of x and y-coordinates.
If we know the location of two points on the line, we can find the equation for the
line.
The dotted green line on the coordinate plane above has a slope of

. The line crosses the y-axis when it has an x-value of 0, so

its y-intercept is at –5. Therefore, the equation of the line is

Finding The Equation Of The Line


[Page 29 of 32]

Task: Find the equation of the line on the coordinate plane below. Type your
answer into the text box, and then click Continue.

The equation is =
We need to find the slope, and then we can find the y-intercept. The slope is

. For every 8 units to the right, the line climbs 15 units upward.
To find the y-intercept, we can move 6 units to the right from the point marked (–
6, –1) toward the y-axis, to determine our y-value. We also move

up as we do so. Moving up and 6 to the right gives us the y-

intercept of .

So the equation of the line is .


Slopes Of Parallel Lines
[Page 30 of 32]

Lines that are parallel have the same slope.


A line with the equation y = 3x + 8 is parallel to the line y = 3x + 5. The first line
crosses the y-axis 3 units above the second, and at every point is 3 units above
the second line.

Slopes Of Perpendicular Lines


[Page 31of 32]

div>

When lines are perpendicular, their slopes are negative reciprocals. A reciprocal
is a fraction in which the numerator and the denominator are reversed.

The reciprocal of is . The reciprocal of –2 is , since –2 is . The negative

reciprocal of is . The negative reciprocal of –2 is .

On the coordinate plane above, if Line 1 has a slope of , Line 2 has a slope of

También podría gustarte